PT3.S2.Q04 - a work of architecture

ChiTownGuyChiTownGuy Alum Member
edited January 2016 in Logical Reasoning 179 karma
I'm having a hard time understanding how this answer choice follows logically from the premises.

Comments

  • runiggyrunruniggyrun Alum Inactive Sage Inactive ⭐
    edited January 2016 2481 karma
    I think this is one of those where formal lawgic might help simplify things. The first sentence can be rewritten like this:
    IF inviting AND functional THEN unobtrusive


    The contrapositive "IF NOT unobtrusive, THEN NOT functional for public use or NOT inviting.

    The conclusion states "They (modern architects) have let their strong personalities take over their work producing buildings that are not functional for public use"
    So: IF personalities take over THEN NOT functional for public use.

    Now, there might be multiple ways in which personalities taking over could lead to unfunctional buildings, but there's a key sentence the author provides to clarify this: architects are specifically violating the principle in the premises above, so we must be triggering the contrapositive - more specifically the IF not unobtrusive then NOT functional for public use (the inviting becomes irelevant, as we are provided with "Not functional for public use". (That is the only thing in the paragraph that can be violated to get to "Not functional for public use").
    That leads us to answer B: IF Strong personalities take over THEN buildings are NOT unobtrusive. (and if not unobtrusive, then not functional for public use).
    A. is wrong because it's an illegal reversal of logic for both premises a and b
    C. is too strong, and shifts from "letting strong personalities take over their work" to merely having a strong personality
    D. Another illegal reversal of logic, this time taking things one step further from unobtrusive to takes second place to environment
    E. Too strong. It can express the personality, as long as it doesn't take over.
  • ChiTownGuyChiTownGuy Alum Member
    179 karma
    I thought the answer was reached along the same lines as you have explained above too originally, then I realized that this is a must be true inference question, and so you the way you have tried to explain it, and the way I was also originally thinking about it is not in line with how an inference question is done. The way that you have explained is is more in line with how a sufficient assumption question is done, where you supply the missing premise to justify the conclusion. Here you are justifying by saying if strong personality, then unobtrusive is added to the premises to justify the conclusion that modern architects have violated the precept in the first sentence. This is where the confusion for me lies. This is a MBT not a sufficient assumption, and the way you are suggesting to solve is a method for sufficient assumption I think.
  • nye8870nye8870 Alum
    1749 karma
    This question has me flummoxed. I mapped this as: (I & FPU) –must-> /O (unobtrusive)
    Now concerning modern architects they: LSPTOW –producing buildings--> /FPU
    By negating a sufficient variable as far as I know you do not get to conclude the negated necessary.
    But (B) says: LSPTOW ----> /FPU ---> O (not not obtrusive aka obtrusive)
    (side note question…Is obtrusive/unobtrusive a binary cut?)
    Any thoughts @c.janson35 ?
  • nye8870nye8870 Alum
    1749 karma
    @runiggyrun said:
    That leads us to answer B: IF Strong personalities take over THEN buildings are NOT unobtrusive. (and if not unobtrusive, then not functional for public use).
    I feel like if unobtrusive fails that means that either I or FPU failed or both....so FPU could stand.
  • nye8870nye8870 Alum
    1749 karma
    @runiggyrun said:
    The first sentence can be rewritten like this:
    a. IF inviting, THEN unobtrusive
    b. IF functional for public use THEN unobtrusive.
    Does this coincide with this lesson?
    http://7sage.com/lesson/advanced-andor-in-sufficient-conditions/
  • ChiTownGuyChiTownGuy Alum Member
    179 karma
    Yeah I'm still confused. The answer to this question to me reads like a sufficient assumption that is supplied in order to make the conclusion follow logically, not (as the question stipulates) a conclusion that follows logically from the premises.
  • runiggyrunruniggyrun Alum Inactive Sage Inactive ⭐
    2481 karma
    @nye8870 Yes, you're right it's an if (A+B) then C, contrapositive if not C then not A or not B. I edited my original answer to reflect this. In either case, the "inviting" part is irrelevant to the argument, since the author tells us the result is buildings that are not functional for public use.
    It's not a nice question, for sure, especially for a 4th question. To me it doesn't feel quite tight enough to be an MBT. I think the key is the sentence that "modern architects have violated this precept". Seeing how the precept only has one necessary condition, unobtrusiveness, that's the one that must have been negated. B links the "personalities taking over" to not being unobtrusive.
  • c.janson35c.janson35 Free Trial Inactive Sage Inactive ⭐
    2398 karma
    The problem with trying to conclude anything from "not functional" is that it is the necessary condition of the contrapositive that you have diagrammed. Nothing can be gleaned from telling us the necessary condition occurs because it can always stand on its own independent of the sufficient.

    I worked through this problem like this:

    If something is a work of architecture that is to be functional and inviting, then it must be unobtrusive.

    We know they have violated this precept. This only occurs when the conditional is a false statement, or has a false truth-value, similar to disproving a general rule. A conditional is false when the sufficient occurs without the necessary, so this is the scenario that the stim describes. Modern architects thusly violate the precept by producing works of architecture (buildings) that are not unobtrusive, which is closest to answer choice B.
  • nye8870nye8870 Alum
    1749 karma
    @c.janson35 wow...thx. That is a strange scenario we do not often see. Would you say this is probably something that won't show up in a modern lsat ?
  • ChiTownGuyChiTownGuy Alum Member
    179 karma
    That actually makes sense to me, thanks!!
  • ChiTownGuyChiTownGuy Alum Member
    179 karma
    Actually that explanation , upon further thought still doesn't make sense to me. If the conditional says IF F + I, then U. And the architects are in violation of that precept, and violation means that they have done the sufficient without doing the necessary, then wouldn't that mean that either they have done F+I and /U or the have done /U and F+I. But the premises say that they have not done F. And /f and /I are each indecently sufficient to satisfy the necessary condition. So How could they have violated F+I then U if the definition of violation is sufficient occurring without the necessary occurring. The answer says that they have done /U, so that would mean that if they were in violation of F+I then U, then they would have to have done BOTH F AND I to be in violation of the precept, but the premises say they have satisfied /F, therefore they have satisfied the necessary condition of the original statement contrapositive.
  • ChiTownGuyChiTownGuy Alum Member
    179 karma
    The answer choice to this question seems to me like it is the invalid argument form #2
  • DumbHollywoodActorDumbHollywoodActor Alum Inactive ⭐
    7468 karma
    I’m not sure what you mean by Sufficient Assumption. There’s not really a conclusion in this stimulus.

    We’re given a principle that if an architectural work is inviting and functional then it is unobtrusive (I and F -->U) Then we’re told that Modern Architects violate this rule. In other words, they deny the relationship ([not (I and F -->U)]. From our lesson in denying a relationship we can infer that some Modern Architects’ work is inviting and functional and NOT unobtrusive (I&Fsome/U). Clearly, modern architects are sometimes not being unobtrusive, hence B.
    @syed.216 said:
    violation means that they have done the sufficient without doing the necessary,
    Not quite. A violation is an intersection, not a new conditional statement. Here’s the lesson.
    http://7sage.com/lesson/how-to-negate-statements-in-english/. The trickiness with answer B is that there is an implied “some”.
    @syed.216 said:
    The answer choice to this question seems to me like it is the invalid argument form #2
    Invalid Argument #2 (or denying the sufficient) refers to conditional statements, but as you can see, because modern architects violate the rule, we’re dealing with a SOME statement instead.
  • ChiTownGuyChiTownGuy Alum Member
    179 karma
    The original statement is a conditional, when negated yields /u then /f or /i. The lesson you referenced is for negating intersection statements (some/most/all). Some one else suggested that the sentence "violation of this precept (i and f then u) would be when they do the sufficient and not the necessary. Also I don't think I've ever seen an answer choice that has an implied some, also I think the negation of the statement would read "some things that are inviting and functional are not unobtrusive. But again, the premises tell us that they are creating buildings that are not functional. Maybe I'm over thinking this but as you can see, now 4 different people have explained this 4 different ways...
  • ChiTownGuyChiTownGuy Alum Member
    179 karma
    Also both powerscore and kaplan suggest that "violation of this precept) means that they are completing a sufficient condition without comepleting a necessary condition. BUt i don't see how that's possible given the correct answer choice. TO do that either you would have F+I then /u, but thats not possible because we are told /F. Also, /u then /f or /I, but again here the necessary condition is satisfied.
  • DumbHollywoodActorDumbHollywoodActor Alum Inactive ⭐
    edited January 2016 7468 karma
    @syed.216 said:
    The original statement is a conditional, when negated yields /u then /f or /i.
    This is the part that I’m trying to tell you is wrong with your reasoning. When you negate a conditional, you end up with an intersectional statement.
    @syed.216 said:
    The lesson you referenced is for negating intersection statements
    No it’s not. Read it again. It’s for negating conditional statements. It literally says "Negating the Conditional Sentence” in bold.
    @syed.216 said:
    Also I don't think I've ever seen an answer choice that has an implied some,
    Sure you have. “people are morons” Am I saying all people are morons in this sentence or just some? Clearly, I could mean “some”.
    @syed.216 said:
    But again, the premises tell us that they are creating buildings that are not functional.
    Because we’re dealing with necessity, some MBT questions will only focus on a small part of the stimulus and the rest will be irrelevant. The fact that modern architects make buildings that are not functional is irrelevant because it’s negating the sufficient condition (just like in LG, the rule falls away) of the principle in the stimulus.

    You’re not overthinking it. You’re just not clear what’s relevant and what isn’t to this particular question, which is understandable because it’s tricky. Hope this clarification helps.
  • c.janson35c.janson35 Free Trial Inactive Sage Inactive ⭐
    2398 karma
    The only thing that matters is the violation of the precept. This is because there is only one way to violate a precept/ disprove a general rule/ offer a counterexampe/ assign a false truth-value to a conditional statement: the sufficient occurs without the necessary. So the first sentence gives us the general rule. The second sentence says this rule was violated. This HAS to mean that the sufficient occurred without the necessary occurring. We don't need any more information that tells us in a specific instance that the sufficient is in fact occurring; it is implied when we are told the rule has been violated. This question is weird in that the last sentence tells us in some instances the sufficient does not occur (not functional), but this does not matter because there must be one instance in which the sufficient did occur and the necessary did not, or else there wouldn't be a way for the conditional statement to be false/ the rule to have been violated.

    For more information, check out the logical implication table here: http://sites.millersville.edu/bikenaga/math-proof/truth-tables/truth-tables.html
  • hihihi9993hihihi9993 Member
    edited February 2018 342 karma

    deleted

Sign In or Register to comment.